suppose you are standing on a train accelerating at 0.30 g . part a what minimum coefficient of static friction must exist between your feet and the floor if you are not to slide?

Answers

Answer 1

When standing on a train accelerating at 0.30 g, there is an effective force acting on you due to the acceleration. This force is equivalent to the force that would be experienced by an object with mass m = your mass under the influence of gravity and this force is resisted by the static friction force:

F = m * a

where a is the acceleration of the train and g is the acceleration due to gravity (approx. 9.81 m/s^2).

To avoid sliding on the floor of the train, the static friction force between your feet and the floor must be greater than or equal to the force due to the acceleration of the train. Therefore, we have:

f_s >= m * a

where f_s is the static friction force.

The maximum static friction force that can act between your feet and the floor is given by:

f_s = μ_s * N

where μ_s is the coefficient of static friction between your feet and the floor, and N is the normal force acting on your feet.

Since you are standing still relative to the train, the normal force acting on your feet is equal to your weight, which we can express as:

N = m * g

Substituting this into the expression for the maximum static friction force, we get:

f_s = μ_s * m * g

Substituting this expression for f_s into the inequality above, we get:

μ_s * m * g >= m * a

Simplifying this expression, we get:

μ_s >= a / g

Substituting a = 0.30 g and g = 9.81 m/s^2, we get:

μ_s >= 0.30

Therefore, the minimum coefficient of static friction that must exist between your feet and the floor to avoid sliding on the train is 0.30.

Learn more about static friction:

https://brainly.com/question/22438157

#SPJ11


Related Questions

you are holding a box of books with flat hands. if you press harder, what happens to the friction force applied by your hands onto the sides of the box?

Answers

When you hold a box of books with flat hands and press harder, the friction force applied by your hands onto the sides of the box will increase.

The force causes motion because if an object is at rest, it remains at rest until acted upon by a force. If the object is in motion, it remains in motion unless acted upon by a force to slow it down, speed it up, or change its direction. So, we have to look at the direction of the force and the motion to understand how the force will affect it. In general, the frictional force opposes motion.

The force of friction is proportional to the force pressing the two surfaces together. In this case, the force pressing the box onto your hands will be greater if you press harder, resulting in a greater frictional force applied by your hands onto the sides of the box, according to Coulomb's laws. Therefore, the friction force will increase when you press harder.

Learn more about friction force at https://brainly.com/question/15122221

#SPJ11

mass than air) at the same temperature. how does this affect the normal-mode frequencies of the pipe?

Answers

When a pipe is filled with a liquid of higher density than air, the frequency of the normal modes of the pipe decreases.

This is due to the fact that the speed of sound is proportional to the square root of the ratio of the bulk modulus to the density of the medium in which it travels.

When the density of the medium inside the pipe increases, the velocity of sound decreases, causing the frequency of the normal modes to decrease.

he wavelength of the sound waves inside the pipe is shortened due to the increase in density, resulting in a lower frequency of the normal modes.

The frequency of the normal modes of a pipe is influenced by a variety of factors, including the diameter and length of the pipe, as well as the speed of sound in the medium inside the pipe. T

he frequency of the normal modes is inversely proportional to the length of the pipe, with longer pipes producing lower frequencies.

In the case of a pipe filled with a liquid of higher density than air, the frequency of the normal modes would be lower than if it were filled with air.

This is because the speed of sound in the liquid would be lower than in air, resulting in a decrease in the frequency of the normal modes.

When a pipe is filled with a liquid of higher density than air, the frequency of the normal modes of the pipe decreases.

This is due to the fact that the speed of sound in the liquid is lower than in air, resulting in a decrease in the frequency of the normal modes.

to know more about frequency refer here:

https://brainly.com/question/14316711#

#SPJ11

A 750-kg roller coaster car drops from rest at a height of 90.0 m along a frictionless track. If the coefficient of kinetic friction due to braking along a horizontal track at the end of the ride is 0.720, over what distance does the car need to brake to come to a complete stop?

Answers

Roller coaster car's mass -
m
=
750

k
g
m=750 kg
Initial height -
h
1
=
90

m
h
1

=90 m
Coefficient of friction -
μ
k
=
0.72
μ
k

=0.72

Two pieces of clay are thrown towards each other. The blue clay has a mass of 2 kg and is traveling at 1.5 m/s east. The red clay has a mass of 1.5 kg and is
traveling at 2.5 m/s west (negative velocity). They stick together after they collide. What is the final velocity of the combined clay pieces after the collision? East
is considered positive direction.

Answers

Explanation:

To solve this problem, we can use the law of conservation of momentum, which states that the total momentum of a system is conserved in the absence of external forces.

Before the collision, the momentum of the blue clay is:

momentum of blue clay = mass of blue clay * velocity of blue clay

= 2 kg * 1.5 m/s = 3 kg*m/s to the east (positive)

Before the collision, the momentum of the red clay is:

momentum of red clay = mass of red clay * velocity of red clay

= 1.5 kg * (-2.5 m/s) = -3.75 kg*m/s to the west (negative)

The total momentum before the collision is:

total momentum before collision = momentum of blue clay + momentum of red clay

= 3 kgm/s - 3.75 kgm/s = -0.75 kg*m/s to the west (negative)

After the collision, the two clays stick together and move as one combined object. Let's assume that the final velocity of the combined clay pieces after the collision is v.

By the law of conservation of momentum, the total momentum after the collision is equal to the total momentum before the collision:

total momentum after collision = total momentum before collision

= -0.75 kg*m/s

The combined mass of the two clays after the collision is:

combined mass = mass of blue clay + mass of red clay

= 2 kg + 1.5 kg = 3.5 kg

Therefore, the final velocity of the combined clay pieces after the collision is:

v = total momentum after collision / combined mass

= (-0.75 kg*m/s) / 3.5 kg

= -0.214 m/s to the west (negative)

Since the negative velocity indicates a direction to the west, the final velocity of the combined clay pieces after the collision is 0.214 m/s to the west.

which of the following will increase the capacitance of a parallel-plate capacitor? (there could bemore than one correct choice.) a) an increase in the charge on the platesb) an increase in the potential difference

Answers

D) introduce a dielectric material between the plates, and E) decrease the separation between the plates will increase the capacitance of a parallel-plate capacitor.

The capacitance of a parallel-plate capacitor is given by the formula:

C = εA/d

where C is the capacitance, ε is the permittivity of free space, A is the area of the plates, and d is the distance between the plates.

From this formula, we can see that the capacitance is directly proportional to the area of the plates and the permittivity of free space, and inversely proportional to the distance between the plates. Therefore, the following changes will increase the capacitance of a parallel-plate capacitor:

D) Introduce a dielectric material between the plates: A dielectric material has a higher permittivity than air, which increases the capacitance of the capacitor.

E) Decrease the separation between the plates: A decrease in the distance between the plates increases the capacitance of the capacitor.

Therefore, the correct choices are D) introduce a dielectric material between the plates, and E) decrease the separation between the plates.

Learn more about Capacitor: https://brainly.com/question/17176550

#SPJ11

a boat moves through the water with two forces acting on it. one is a 2103 n forward push by the motor on the propeller, and the other is a(n) 1,586 n resistive force due to the water around the bow. what is the acceleration of the 1193 kg boat?

Answers

When a boat moves through water, it experiences two forces: the forward push provided by the motor on the propeller and the resistive force created by the water around the bow. The acceleration of the 1193 kg boat is 0.404 m/s².

The net force acting on the boat can be calculated by subtracting the resistive force from the forward force:

F_net = F_forward - F_resistiveF_net = 2103 N - 1586 NF_net = 517 N

The acceleration of the boat can be calculated using the formula: a = F_net/m Where F_net is the net force acting on the boat, and m is the mass of the boat. Substituting the values we know, a = 517 N / 1193 kg a = 0.404 m/s²Therefore, the acceleration of the 1193 kg boat is 0.404 m/s².

To know more about acceleration, refer here:

https://brainly.com/question/28587095#

#SPJ11

an electron moves at a speed of 3x10^4 m/s parallel to the uniform magnetic field of 0.4t. it experiences a force of what magnitude?

Answers

The magnitude of the force experienced by the electron is 1.92 x 10^-14 N.

The force experienced by a charged particle moving in a magnetic field is given by the formula,

F = qvB

where F is the force on the particle, q is the charge of the particle, v is the velocity of the particle, and B is the magnetic field.

In the given problem, the electron is moving parallel to the magnetic field, so the angle between the velocity vector and the magnetic field vector is 0 degrees. Therefore, the sine of the angle is 0, and the force experienced by the electron is simply,

F = qvB

where q is the charge of the electron (-1.6 x 10^-19 C), v is the speed of the electron (3 x 10^4 m/s), and B is the magnetic field (0.4 T).

Substituting the given values,

F = (-1.6 x 10^-19 C) * (3 x 10^4 m/s) * (0.4 T)

F = -1.92 x 10^-14 N

To know more about magnetic field, here

brainly.com/question/28246026

#SPJ4

jasmin, a cyclist, accelerates from rest. after 8 s, the wheels have made 3 revolutions. (a) what is the angular acceleration of the wheels? (b) what is the angular velocity of the wheels after 8 s?

Answers

a. The angular acceleration of the wheels is 0.2945 rad/s². b. The angular velocity of the wheels after 8 seconds is 2.3560 rad/s.

Calculation:

a. The formula for angular acceleration is: α = (ω2 - ω1) / (t2 - t1) Whereα is angular acceleration, ω2 is final angular velocity, ω1 is initial angular velocity, t2 is final time, t1 is initial time. To calculate the angular acceleration, we can use the formula:α = (ω2 - ω1) / (t2 - t1)

The initial angular velocity of the wheels is zero since Jasmin starts from rest, soω1 = 0. We know that the wheels make 3 revolutions after 8 seconds, so the final angular velocity can be calculated as follows: ω2 = (3 revolutions / 8 s) x (2π radians / 1 revolution) = 2.3562 rad/s

Therefore,α = (2.3562 rad/s - 0 rad/s) / (8 s - 0 s) = 0.2945 rad/s². The angular acceleration of the wheels is 0.2945 rad/s².

b. To calculate the angular velocity of the wheels after 8 seconds, we can use the formula:ω = ω1 + αtWhereω is angular velocity,ω1 is initial angular velocity,α is angular acceleration, t is time. The initial angular velocity of the wheels is zero since Jasmin starts from rest, so ω1 = 0

We have already calculated the angular acceleration to be 0.2945 rad/s², and we know that the time is 8 seconds, soω = ω1 + αt = 0 + (0.2945 rad/s²) x (8 s) = 2.3560 rad/s. Therefore, the angular velocity of the wheels after 8 seconds is 2.3560 rad/s.

To know more about angular acceleration, refer here:

https://brainly.com/question/29428475#

#SPJ11

when a toolbox weighing 5 newtons is resting on the ground next to a sawhorse, how much potential energy does it have?

Answers

The potential energy of a toolbox weighing 5 newtons is zero.

The potential energy of a toolbox weighing 5 newtons depends on its height relative to the ground.

Potential energy (PE) is equal to the mass of the object (m) multiplied by the acceleration due to gravity (g) multiplied by its height (h): PE = mgh.

Therefore, the potential energy of the toolbox is equal to 5*9.8*h (where h is the height of the toolbox above the ground).

Assuming that the toolbox is resting on the ground, it has zero potential energy since its height is zero. If the toolbox is lifted above the ground, however, then it will have a greater potential energy.

For example, if the toolbox is lifted to a height of 10 meters above the ground, then it will have a potential energy of 490 joules (5*9.8*10).

The potential energy of the toolbox when it is placed next to the sawhorse, the height of the sawhorse needs to be taken into consideration.

If the sawhorse is higher than the ground, then the toolbox will have a greater potential energy since it will be located at a greater height above the ground.

If the sawhorse is lower than the ground, then the toolbox will have a lesser potential energy than when it is resting on the ground.

The potential energy of a toolbox weighing 5 newtons when placed next to a sawhorse depends on the height of the sawhorse relative to the ground.

If the sawhorse is higher than the ground, then the toolbox will have a greater potential energy, and if it is lower than the ground, then the toolbox will have a lesser potential energy.

to know more about potential energy refer here:

https://brainly.com/question/24284560#

#SPJ11

you are standing on a scale in an elevator. suddenly you notice your weight increases. what do you conclude?

Answers

When standing on a scale in an elevator, if one notices an increase in their weight, it means that: the elevator is accelerating upwards.

This is due to the fact that the scale underfoot has to counter the upward acceleration of the elevator, which causes the weight measured on the scale to increase. The scale measures the normal force, which is the weight being exerted on the scale, which is equal to the mass of the individual multiplied by the gravitational acceleration on the surface of the earth.

This can be represented by the formula: W = mg,

where W is the weight, m is the mass of the object and g is the gravitational acceleration.

When the elevator is stationary or moving at a constant velocity, the gravitational acceleration is the same as the normal force and the weight of the individual remains constant. However, when the elevator begins to accelerate upwards, the normal force exerted by the scale must increase to counter the upward acceleration of the elevator.

This causes an increase in weight measured on the scale. Therefore, if one notices an increase in their weight while standing on a scale in an elevator, it indicates that the elevator is accelerating upwards.

To know more about acceleration refer here:

https://brainly.com/question/30660316#

#SPJ11

a box is given a push so that it slides across the floor. how far will it go, given that the coefficient of kinetic friction is 0.11 and the push imparts an initial speed of 3.8 m/s ?

Answers

The box will slide a distance of 6.96 m before coming to a stop due to the force of kinetic friction.

To determine how far the box will slide on the floor after it is given a push with an initial speed of 3.8 m/s, we need to use the equations of motion for constant acceleration. The force of kinetic friction acting on the box will cause it to decelerate, eventually coming to a stop.

The distance traveled by the box can be found using the equation:

d = [tex](v_i^2 - v_f^2) / (2 * a)[/tex]

where d is the distance traveled, v_i is the initial speed, v_f is the final speed (which is zero since the box comes to a stop), and a is the deceleration caused by the force of kinetic friction.

The deceleration can be found using the equation:

a = -F[tex]_friction / m[/tex]

where Ffriction is the force of kinetic friction and m is the mass of the box.

Assuming a mass of 5 kg for the box and a coefficient of kinetic friction of 0.11, the force of kinetic friction can be found using the equation:

F_friction = friction coefficient * F_normal

where F_normal is the normal force (equal to the weight of the box) and the friction coefficient is a dimensionless quantity that depends on the nature of the contact surface.

The weight of the box is:

Fweight = m * g

where g is the acceleration due to gravity (9.81 m/s²).

Therefore, the force of kinetic friction is:

F_friction = (0.11) * (5 kg * 9.81 m/s²) = 5.40 N

Using the equation for deceleration, we get:

a = -Ffriction / m = -(5.40 N) / (5 kg) = -1.08 m/s²

Finally, we can use the equation for distance traveled to find the distance the box will slide:

d = [tex](v_i^2 - v_f^2) / (2 * a)[/tex] =[tex](3.8 m/s)^2 / (2 * 1.08 m/s^2)[/tex] = 6.96 m

Learn more about kinetic: https://brainly.com/question/26472013

#SPJ11

an object is located in water 30 cm from the vertex of a convex surface made of plexiglas with a radius of curvature of 80 cm. where does the image form by refraction and what is its magnification? nwater

Answers

The image formed by refraction is at a distance of 120cm behind the lens and its magnification is 4.

The image formation by refraction and magnification of an object in water 30 cm from the vertex of a convex surface made of plexiglass with a radius of curvature of 80 cm can be calculated using the following steps:

1. Determine the object's distance from the lens. Object distance (u) = -30 cm. (negative sign as per the convention of the mirror)

2. Determine the focal length of the lens using the formula:

f = R/2 where, f = focal length of the lens, R = radius of curvature of the lens.

So, f = 80/2 = 40 cm.

3. Use the mirror formula to determine the image distance from the surface:

1/f = 1/v + 1/u where,v = image distance from the surface.

Substituting the given values (with proper sign convention), we get:

(-1/40) = 1/v + (-1/30)

Solving for v, we get:

v = 120 cm.

5. Use the magnification formula to determine the magnification of the image:

m = -(v/u)

where,m = magnification, v = 120 cm, u = 30 cm

Therefore,m = -(120/-30) = 4

Therefore, the image will form at a distance of 120 cm from the lens on the water side of the lens and is magnified by a factor of 4.

To know more about refraction, refer here:

https://brainly.com/question/26528081#

#SPJ11

a syringe containing an incompressible fluid is oriented vertically and the plunger slowly depressed. at which point is the kinetic energy the lowest?

Answers

The point at which the kinetic energy is lowest is 3 in the syringe containing an incompressible fluid that is vertically oriented and the plunger is slowly depressed.


What is kinetic energy?

The kinetic energy of an object is the energy it has due to its motion. When an object is in motion, it has kinetic energy. It is a scalar quantity that is proportional to the mass of the object and the square of its velocity. The formula for kinetic energy is given as follows:

                                KE = 1/2mv²

Where m is the mass of the object and v is its velocity.

Points 1 and 2 have higher kinetic energy because the incompressible fluid is still being compressed in the syringe. Point D is incorrect because the kinetic energy of the incompressible fluid is not the same at all three points. Point E is incorrect because enough information has been provided. Therefore, when a syringe containing an incompressible fluid is vertically oriented and the plunger is slowly depressed, the kinetic energy is lowest at point 3.

Learn more about kinetic energy at https://brainly.com/question/25959744

#SPJ11

a 67.97 kg student is standing atop a spring in an elevator that is accelerating upward at 3.66 m/s2. the spring constant is 2658 n/m. by how much is the spring compressed?

Answers

The spring is compressed by 0.70 m.

The 67.97 kg student standing atop a spring in an elevator that is accelerating upward at 3.66 m/s2 has a spring constant of 2658 n/m. The amount the spring is compressed can be calculated using the formula:

F = kx,

where F is the force, k is the spring constant, and x is the amount of compression.

Therefore, we can calculate the amount of compression in the spring as follows:

x = F/k = (67.97 kg * 3.66 m/s2)/2658 N/m = 0.70 m

Therefore, the spring is compressed by 0.70 m.

To know more about Spring compression, refer here:

https://brainly.com/question/12278394#

#SPJ11

a 5 kg toy train car is connected to a 3 kg toy train car. the 3 kg car is given an external force of 16 n. what is the tension in the rope connecting the cars?

Answers

A 5 kg toy train car is connected to a 3 kg toy train car. the 3 kg car is given an external force of 16 n. the tension in the rope connecting the two cars is 29 N.

The tension in the rope connecting two toy train cars A toy train car with a mass of 5 kg is connected to a toy train car with a mass of 3 kg. An external force of 16 N is applied to the 3 kg car.

Tension in the rope between the two toy cars is what we need to calculate. According to Newton’s 2nd law, force equals mass multiplied by acceleration. If the two cars are moving in the same direction with the same acceleration, the tension in the rope can be calculated as follows:

Force acting on the two cars is the external force that is applied on the 3 kg car which is equal to 16 N. In this case, both cars will have the same acceleration.

for such more question on tension

https://brainly.com/question/24994188

#SPJ11

while a car travels around a circular track at a constant speed, its a) acceleration is non-zero and along the path b) acceleration is non zero and inward toward the center c) acceleration is zero d) acceleration is non-zero and outward from the center

Answers

While a car travels around a circular track at a constant speed, its (d) acceleration is non-zero and outward from the center.

When a car travels around a circular track at a constant speed, it is constantly changing direction, and therefore, constantly accelerating. This acceleration is known as centripetal acceleration and is directed towards the center of the circle.

However, according to Newton's third law, every action has an equal and opposite reaction. In this case, the car also experiences an equal and opposite acceleration, known as the centrifugal acceleration, which is directed outward from the center of the circle.

This is the non-zero acceleration experienced by the car, and it acts to counterbalance the centripetal acceleration, keeping the car moving in a circular path.

Therefore, the correct answer is (d) acceleration is non-zero and outward from the center.

For more questions like Acceleration click the link below:

https://brainly.com/question/12550364

#SPJ11

Two weights are connected by a massless wire and pulled upward with a constantspeed of 1.50 m/s by a vertical pull P. The tension in the wire is T(see figure). Whichone of the following relationships between Tand Pmust be true?A)TB)T=PC)P+T=125ND)P=T+25N

Answers

Two weights are connected by a massless wire and pulled upward with a constant speed of 1.50 m/s by a vertical pull P. The tension in the wire is T The relationship between T and P is that T = P + 125N, which is equivalent to answer choice D. The correct answer is D) P=T+25N.

This can be determined by analyzing the forces acting on the system. Since the weights are being pulled upward at a constant speed, the net force acting on them must be zero.
The forces acting on the weights are their respective weights (mg), where m is the mass of the weight and g is the acceleration due to gravity, and the tension in the wire (T). The vertical pull P also acts on the system.
Using Newton's second law (F=ma) and setting the net force equal to zero, we can write:
T - m1g - m2g - P = 0
Solving for T, we get:
T = m1g + m2g + P
Substituting in the given values of m1, m2, and g, we get:
T = 50N + 75N + P
Simplifying, we get:
T = P + 125N

For more such questions on tension

https://brainly.com/question/30592298

#SPJ11

one cycle of the power dissipated by a resistor ( ) is given by this periodic signal repeats in both directions of time. what is the amplitude of the pwm voltage signal applied across the 500- resistor

Answers

The maximum amplitude of the PWM voltage signal applied across the 500-ohm resistor is: Vmax=2*I*R=500*I

The power dissipated by a resistor during one cycle is given by the periodic signal. The PWM voltage signal applied across a 500 Ω resistor is analyzed in this question. The amplitude of the signal is determined below.

Pulse Width Modulation is the PWM. It's a process for varying the pulse width of a square wave, which changes the percentage of time the signal is high to low. The pulse width can be varied to create the desired output signal level. It is frequently utilized in applications where analog signals are required, including control systems, power supplies, and audio systems. The maximum voltage Vm of the PWM voltage signal can be found by calculating the RMS value of the pulse. The root-mean-square value is the square root of the mean of the square of the signal over a given period. If we use a pulse that has a duty cycle of 50%, this formula simplifies to: Vmax=Vm+0.5Vdc where Vdc is the average value of the pulse.

The maximum amplitude can be determined using this formula: Vmax=I*R where I is the current and R is the resistance. The current flowing through the resistor is proportional to the voltage applied to it, and the voltage is proportional to the duty cycle of the PWM signal, which varies from 0 to 1. Thus, the voltage applied to the resistor is proportional to the duty cycle and can be expressed as: V=Vmax*D where D is the duty cycle. Thus, the amplitude of the PWM voltage signal applied across a 500-ohm resistor is: Vmax=2*I*R=500*I. Using this equation, we can determine the maximum amplitude of the PWM voltage signal applied across the 500-ohm resistor.

For more such questions on Pulse Width Modulation.

https://brainly.com/question/29358007#

#SPJ11

when einstein's theory of gravity (general relativity) gained acceptance, it demonstrated that newton's theory had been?a. wrongb. incompletec. really only guess

Answers

When Einstein's theory of gravity (general relativity) gained acceptance, it demonstrated that Newton's theory had been (b) incomplete.

Newton's theory of gravity is a law that governs the behavior of objects. The formula [tex]F = \frac {G m_1  m_2}{ d^2}[/tex] explains the force of gravity between two objects, where F is the force of gravity, G is the universal gravitational constant, m1 is the mass of one object, m2 is the mass of another object, and d is the distance between the centers of the two objects. This formula shows that gravity decreases as distance increases.

Einstein's theory of gravity (general relativity): It is a theoretical framework proposed by Albert Einstein in 1915. It combines special relativity and Newton's law of universal gravitation. General relativity is based on the notion that gravitation is not a force acting between two masses but rather a curvature of spacetime created by the presence of massive objects. It differs from Newton's law of universal gravitation, which states that gravitation is caused by an attractive force acting between two masses.

When Einstein's theory of gravity (general relativity) gained acceptance, it demonstrated that Newton's theory had been incomplete. Therefore the correct answer is b.

Learn more about Albert Einstein:

https://brainly.com/question/1275198

#SPJ11

a 5-kg shark swimming at 1 m/s swallows an absent-minded 1-kg fish swimming toward it at 4 m/s. the speed of the shark after his meal is

Answers

The speed of the shark after it swallows the fish is calculated using the conservation of momentum principle. The total momentum before the collision is 5 kg * 1 m/s + 1 kg * 4 m/s = 9 kg * m/s. The total momentum after the collision is 5 kg * v, where v is the speed of the shark after the collision. Therefore, v = 9/5 m/s = 1.8 m/s. Thus, the speed of the shark after it swallows the fish is 1.8 m/s.

The speed of the shark after it has swallowed the 1-kg fish swimming toward it at 4 m/s is 3 m/s. This can be determined by conservation of momentum. Momentum is a vector quantity, meaning that the direction of the momentum must also be taken into account.

In this situation, the momentum of the shark before it swallows the fish is 5 kg⋅m/s due to its velocity of 1 m/s. After the shark has eaten the fish, the momentum is 6 kg⋅m/s due to the addition of the fish's momentum of 4 kg⋅m/s. Since momentum is conserved, the momentum of the shark after eating the fish is the same as the momentum of the shark before eating the fish. Since the mass of the shark does not change, the velocity must change to balance out the difference in momentum. This means that the velocity of the shark after eating the fish is 3 m/s.

For more equations related to momentum.

https://brainly.com/question/30677308

#SPJ11

logs sometimes float vertically in a lake because one end has become water-logged and denser than the other. what is the average density of a uniform-diameter log that floats with 20.0% of its length above water?

Answers

Uneven-diameter logs that float with 20.0% of their length above water have an average density of 0.8g/cm3. The density is the proportion of weight to capacity.

An item it's far less compact that liquid may be supported up liquid water, and hence it floats. More dense objects can sink when submerged in water. Less dense logs float whereas more thick logs sink. A body can change its condition of rest or motion by the application of force

Instead of obliquely reading from either the side, read the scale stick straight from of the end of both the log. → The diameter of a log is only ever calculated within the bark. Employ a log measuring rod to determine the log's small end's "diameter from within bark," also known as "d.i.b."

To know more about force click here

brainly.com/question/29044739

#SPJ4

a 6 mf capacitor, a 10 mf capacitor, and a 16 mf capacitor are connected in parallel. what is their equivalent capacitance?

Answers

The equivalent capacitance of a 6 mF capacitor, a 10 mF capacitor, and a 16 mF capacitor connected in parallel is: 32 mF

This is because when capacitors are connected in parallel, their total capacitance is equal to the sum of their individual capacitances. The formula for calculating the equivalent capacitance (C) of capacitors connected in parallel is: C = C1 + C2 + C3 + ... In this example, C = 6 mF + 10 mF + 16 mF = 32 mF.

Capacitors are electrical components that store energy in the form of an electric field between two conductors (plates). When capacitors are connected in parallel, the electric field between the plates of each capacitor is the same, but the overall capacitance is increased due to the combined plate area of all the capacitors.

This increase in plate area is why the equivalent capacitance of the three capacitors in this example is 32 mF, which is larger than any of the individual capacitances.

To know more about capacitors refer here:

https://brainly.com/question/17176550#

#SPJ11

a negatively charged point particle is placed initially at rest in a uniform electric field as a result of being placed in the electric field which direction will it move

Answers

When a negatively charged point particle is placed initially at rest in a uniform electric field, it will move towards the direction of the electric field.

An electric field is a vector field that represents the force exerted by charged particles over each other. It is generated by charges, and it affects other charged particles that are in the space around it. The direction of the electric field is given by the direction of the force that is experienced by a small positive test charge placed in that field. If the force on the test charge is towards the positive charge that creates the field, the electric field will point towards the positive charge. If the force on the test charge is towards the negative charge that creates the field, the electric field will point towards the negative charge.

When a negatively charged particle is placed in the electric field, it experiences a force in the direction opposite to the direction of the electric field,  this is because the negatively charged particle is attracted towards the positively charged particles that generate the field, and so it moves towards them. Therefore, the negatively charged particle moves towards the direction of the electric field. When a positively charged particle is placed in the electric field, it experiences a force in the direction of the electric field. This is because the positively charged particle is attracted towards the negatively charged particles that generate the field, and so it moves towards them. Therefore, the positively charged particle moves towards the direction opposite to the direction of the electric field.

Learn more about electric field at:

https://brainly.com/question/4440057

#SPJ11

pry on the power steering reservoir to adjust the tension of the power steering belt. true or false?

Answers

The statement "pry on the power steering reservoir to adjust the tension of the power steering belt" is: false.

The tension of the power steering belt is adjusted by adjusting the position of the power steering pump. There is a tension adjustment bolt on the power steering pump that is used to adjust the tension of the power steering belt. The adjustment bolt should be turned clockwise or counterclockwise to adjust the tension of the belt.

A belt tension gauge may be used to ensure that the belt is properly tensioned. A pry bar should not be used on the power steering reservoir to adjust the tension of the power steering belt. This could cause damage to the reservoir or other components of the power steering system. The reservoir should be inspected for damage or leaks, but it should not be used to adjust the tension of the belt.

In summary, the tension of the power steering belt should be adjusted by adjusting the position of the power steering pump, not by prying on the power steering reservoir.

To know more about tension refer here:

https://brainly.com/question/29869473#

#SPJ11

engineers at the university of texas at austin are developing an advanced locomotive propulsion system that uses a gas turbine and perhaps the largest highspeed flywheel in the world in terms of the energy it can store. the flywheel can store 4.8 * 108 j of energy when operating at its maximum rotational speed of 15,000 rpm. at that rate, the perimeter of the rotor moves at approximately 1,000 m/s. determine the radius of the flywheel and its rotational inertia.

Answers

The radius of the flywheel and its rotational inertia will be 0.64m and 389kgm² respectively.

What is the rotational inertia?

Rotational inertia, also known as moment of inertia, is a measure of an object's resistance to rotational motion. It is similar to the concept of mass in linear motion. Just as mass is a measure of an object's resistance to linear motion, the moment of inertia is a measure of an object's resistance to rotational motion.

The moment of inertia of an object depends on its shape and mass distribution. Objects with more mass distributed farther from the axis of rotation have a higher moment of inertia than objects with the same mass but a more compact distribution of mass. The moment of inertia is measured in units of kilograms square meters (kg m²) in the SI system.

The radius will be:

= 1000 / 15000(2πrad / 60)

= 0.64m

The inertia will be:

= 2(4.8 × 10^8) / 100 (2π/60)

= 389kgm²

Learn more about inertia on

https://brainly.com/question/1830739

#SPJ1

most of the mass of the solar system is located in which of the following? responses sun sun jupiter jupiter comets comets earth

Answers

Most of the mass of the solar system is located in the Sun. The Sun accounts for over 99% of the total mass of the solar system, with the remaining mass distributed among the planets, asteroids, comets, and other objects.

The solar system is a collection of objects that orbit around the Sun. It consists of the Sun, eight planets and their natural satellites, dwarf planets, asteroids, comets, and other small bodies. The eight planets, listed in order from the Sun, are Mercury, Venus, Earth, Mars, Jupiter, Saturn, Uranus, and Neptune.

The Sun is at the center of the solar system and contains more than 99% of the mass of the solar system. It is a giant ball of gas, mostly hydrogen, and helium, and is the source of heat and light for the entire solar system.

Learn more about the Sun:

https://brainly.com/question/15837114

#SPJ11

a rising parcel of unstable air a rising parcel of unstable air can rise well into the mesosphere. cannot rise very far above the tropopause. can eventually escape into space. will not be slowed by entrainment.

Answers

A rising parcel of unstable air is an air mass that is warmer than the surrounding air and is therefore buoyant. It can rise until it reaches an area where its temperature is the same as the surrounding air, the tropopause.

The tropopause is the boundary between the troposphere (the lowest part of the atmosphere) and the stratosphere (the next layer of the atmosphere).

At this level, the air is very stable and so the air parcel cannot rise any further.

The air parcel may eventually escape into space, however it will not be slowed by entrainment, the process by which the parcel loses energy and slows down due to friction.

As the parcel rises, the atmospheric pressure decreases and the temperature increases due to the decrease in air density.

As it rises further, the air pressure decreases until it reaches the tropopause, where it then plateaus.

Once the air reaches the tropopause, it has reached a level of equilibrium and can no longer rise further as the temperature and pressure remain constant.

The tropopause also acts as a barrier to air moving from the stratosphere to the troposphere.

This is due to the temperature inversion that occurs when the temperature in the troposphere decreases with altitude while the temperature in the stratosphere increases with altitude.

This inversion creates a strong stratospheric temperature gradient, making it difficult for air to move between the two layers.

A rising parcel of unstable air can rise well into the mesosphere but cannot rise very far above the tropopause.

The tropopause acts as a barrier to air moving between the troposphere and the stratosphere due to its temperature inversion, and the air parcel may eventually escape into space without being slowed by entrainment.

to know more about buoyant refer here:

https://brainly.com/question/21990136#

#SPJ11

Mercury has a mass of 3.29E23 kg and a radius of 2.44E6 m. Venus has a mass of 4.87E24 kg and a radius of 6.05E6 m. The gravitational field near the surface of Mercury is? N/kg. The gravitational field near the surface of Venus is? N/kg.

Answers

Gravitational field near the surface of Mercury is approximately 3.7 N/kg

Gravitational field near the surface of Venus is approximately 8.87 N/kg.

Gravitational field near the surface of Mercury and Venus, we can use the formula:

gravitational field (g) = (G * M) / R^2

where G is the gravitational constant (6.67430 × 10^(-11) m^3 kg^(-1) s^(-2)), M is the mass of the planet, and R is the radius of the planet.

For Mercury:
M = 3.29E23 kg
R = 2.44E6 m

g = (6.67430 × 10^(-11) m^3 kg^(-1) s^(-2) * 3.29E23 kg) / (2.44E6 m)^2
g ≈ 3.7 N/kg

For Venus:
M = 4.87E24 kg
R = 6.05E6 m

g = (6.67430 × 10^(-11) m^3 kg^(-1) s^(-2) * 4.87E24 kg) / (6.05E6 m)^2
g ≈ 8.87 N/kg

So, the gravitational field near the surface of Mercury is approximately 3.7 N/kg, and the gravitational field near the surface of Venus is approximately 8.87 N/kg.

learn more about 'gravitation':https://brainly.com/question/31276986

#SPJ11

we want to lift a load of 200 lb with an overhead system using pulleys that have an efficiency of 0.9. if we can provide a maximum input force of 103 lb, what is the minimum number of pulleys that we need?

Answers

We need at least one pulley to lift the load of 200 lb with an overhead system using pulleys that have an efficiency of 0.9, given that we can provide a maximum input force of 103 lb.

Assuming that the weight of the pulleys and the rope is negligible, we can use the formula,

Load = (Input Force / Efficiencies) ^ Number of Pulleys

where Load is the weight of the load we want to lift, Input Force is the force we apply to the system, Efficiency is the efficiency of each pulley, and Number of Pulleys is the number of pulleys we need.

Plugging in the given values,

200 lb = (103 lb / 0.9) ^ Number of Pulleys

Simplifying the equation,

Number of Pulleys = log (base 2) (200 / (103/0.9))

Number of Pulleys = log (base 2) (200 x 0.9 / 103)

Number of Pulleys = log (base 2) 1.983495

Number of Pulleys = 1

To know more about force, here

brainly.com/question/3854207

#SPJ4

c. what will be the charges of the spheres in fractions of after connection? how does the total charge of the two spheres after the connection compare to the initial charge of the left sphere?

Answers

The charges of the spheres after connection will be the same as the charge of the left sphere. The total charge of the two spheres after connection is equal to the initial charge of the left sphere.

To understand this, it is important to know that electric charge is a conserved quantity. This means that the net charge of a system cannot change. Therefore, if two objects with opposite charges (like the two spheres) are connected, the charges of the two objects will become equal and the total charge of the two spheres will remain the same as the initial charge of the left sphere.
To further understand this concept, consider two spheres with opposite charges. If the two spheres are not connected, then the total charge of the two spheres is equal to the sum of the charges of each sphere. However, if the two spheres are connected, the net charge of the system cannot change. Therefore, the charge of each sphere will become equal and the total charge of the two spheres after the connection will remain the same as the initial charge of the left sphere.

For more such questions on Electric charge.

https://brainly.com/question/12321627#

#SPJ11

Other Questions
A ____ specifies which subjects and objects users or groups can access. What is a disadvantage to developing immunity by being exposed to the bacteria that cause a pertussis? A Rubiks Cube is a game where the person tries to get each surface of a cube to be one color. The Rubiks Cube is made up of smaller unit cubes stacked together.What is the volume, in cubic units, of the Rubiks Cube? g suppose the interest rate increases by 1 point in november of 2009. what effect will that change have on dhomes? why did mr winkle offer sam two coats and five shillings Which of the following quantitative risk analysis techniques is used to determine which risks have the most powerful impact on the project, with results displayed in the form of a tornado diagram?A. Failure Mode and Effects Analysis (FMEA)B. Expected Monetary ValueC. Sensitivity AnalysisD. Simulation Techniques such as Monte Carlo Analysis combined with a high tide on september 16, 2020 by how many feet did that storm surge submerge the lowest parts of pensacola beach? enter a number (no words) in the blank. on may 1, widdle corporation issued 10,000 shares of its $10 par common stock to saddles for a tract of land. the stock had a fair market value of $15 per share on this date. according to saddles' last property tax record, the land was valued at $82,000. what amount should widdle record as an increase in paid-in capital? a.$50,000 b.$82,000 c.$18,000 d.$32,000 In a restaurant, there are 5 managers, 15 servers, 10 cooks and 15 other personnel. If a person is selected at random, what is the probability that the person is either a manager or a cook? Which recursive formula would produce the sequence-44, -46,-48, ... almost all marketing research projects should start with a literature search. group of answer choices true false which of the following is an example of racial profiling? group of answer choices henry, a mexican american, is appointed as the brand ambassador of a motorbike company. zayn, an arab american, is stopped at a turnpike on suspicion of possessing armed weapons. kendrick, an african american, is recruited as a pilot for a commercial airliner. andrew, a white american, is issued a speeding ticket for going ten miles over the limit. Gamma Electronics is considering the purchase of testing equipment that will cost $500,000 to replace old equipment. Assume the new machine will generate after-tax savings of $250,000 per year over the next four years. If the firm has a 15% cost of capital, what's the discount payback period of the investment?a. 2.0 yearsb. 1.5 yearsc. 2.4 yearsd. 2.6 years the nurse is caring for a child recently fitted with braces on both legs due to cerebral palsy (cp). what would the nurse emphasize in the discharge teaching? how many springs does it take to model the thermal energy of diamond and how many for solid argon? explain/justify your answer using appropriate components of the particle model of thermal energy and/or previous models we have used. "In a grove" by ryunosukeIf they all can't be telling the truth, then could it be that the events unfolded in a certain way? And that the way they did, cannot be determined by the evidence we are given? if you click the hide folders button in the save as dialog box, the box collapses and hides the . Which equation has the same solution as x ^2 12x16=2? the flow of funds from into the financial system makes it possible for government and firms to borrow. answer unselected government and firms unselected households unselected investment banks their partnership agreement calls for farmer to receive a $70,000 per year salary allowance. the remaining income or loss is to be divided equally. assuming net loss for the current year is $15,000, the journal entry to allocate the net loss is: